Calculating $sumlimits_{n=1}^inftyfrac{{1}}{n+3^n} $












7












$begingroup$


I was able to prove this sum



$$sum_{n=1}^inftyfrac{{1}}{n+3^n}$$



is convergent through the comparison test but I don't get how to find its sum.










share|cite|improve this question











$endgroup$








  • 2




    $begingroup$
    @FordDavis Ask your teacher about the question again. It's quite unlikely they expect for you to find the sum. Or maybe the question was $frac{1}{n3^n}$, which has a much nicer answer.
    $endgroup$
    – Toby Mak
    Dec 2 '18 at 0:21








  • 2




    $begingroup$
    What about this one: Is there any non-zero function $f(n) $ such that $sum_{n=1}^infty frac{1} {f(n) +3^n}$ has an okay closed form?
    $endgroup$
    – Zacky
    Dec 2 '18 at 0:22








  • 1




    $begingroup$
    @Zacky There are some trivial choices like $f(n) = 2^n - 3^n$.
    $endgroup$
    – Daniel
    Dec 2 '18 at 0:33






  • 1




    $begingroup$
    There's something odd about this assignment. Part b) can be summed in closed form, but it takes techniques way beyond calc 2. Are you perhaps supposed to approximate the sums. I definitely second @TobyMak's advice: ask your teacher about it.
    $endgroup$
    – saulspatz
    Dec 2 '18 at 0:33






  • 3




    $begingroup$
    As noted by Carl, this question already exists ... math.stackexchange.com/q/1672121/442 but even that has no answer in nearly 3 years. No answer = cannot mark this as a duplicate.
    $endgroup$
    – GEdgar
    Dec 2 '18 at 1:14


















7












$begingroup$


I was able to prove this sum



$$sum_{n=1}^inftyfrac{{1}}{n+3^n}$$



is convergent through the comparison test but I don't get how to find its sum.










share|cite|improve this question











$endgroup$








  • 2




    $begingroup$
    @FordDavis Ask your teacher about the question again. It's quite unlikely they expect for you to find the sum. Or maybe the question was $frac{1}{n3^n}$, which has a much nicer answer.
    $endgroup$
    – Toby Mak
    Dec 2 '18 at 0:21








  • 2




    $begingroup$
    What about this one: Is there any non-zero function $f(n) $ such that $sum_{n=1}^infty frac{1} {f(n) +3^n}$ has an okay closed form?
    $endgroup$
    – Zacky
    Dec 2 '18 at 0:22








  • 1




    $begingroup$
    @Zacky There are some trivial choices like $f(n) = 2^n - 3^n$.
    $endgroup$
    – Daniel
    Dec 2 '18 at 0:33






  • 1




    $begingroup$
    There's something odd about this assignment. Part b) can be summed in closed form, but it takes techniques way beyond calc 2. Are you perhaps supposed to approximate the sums. I definitely second @TobyMak's advice: ask your teacher about it.
    $endgroup$
    – saulspatz
    Dec 2 '18 at 0:33






  • 3




    $begingroup$
    As noted by Carl, this question already exists ... math.stackexchange.com/q/1672121/442 but even that has no answer in nearly 3 years. No answer = cannot mark this as a duplicate.
    $endgroup$
    – GEdgar
    Dec 2 '18 at 1:14
















7












7








7


4



$begingroup$


I was able to prove this sum



$$sum_{n=1}^inftyfrac{{1}}{n+3^n}$$



is convergent through the comparison test but I don't get how to find its sum.










share|cite|improve this question











$endgroup$




I was able to prove this sum



$$sum_{n=1}^inftyfrac{{1}}{n+3^n}$$



is convergent through the comparison test but I don't get how to find its sum.







calculus sequences-and-series convergence summation






share|cite|improve this question















share|cite|improve this question













share|cite|improve this question




share|cite|improve this question








edited Dec 2 '18 at 0:51









David G. Stork

10.2k21332




10.2k21332










asked Dec 2 '18 at 0:05









Ford DavisFord Davis

443




443








  • 2




    $begingroup$
    @FordDavis Ask your teacher about the question again. It's quite unlikely they expect for you to find the sum. Or maybe the question was $frac{1}{n3^n}$, which has a much nicer answer.
    $endgroup$
    – Toby Mak
    Dec 2 '18 at 0:21








  • 2




    $begingroup$
    What about this one: Is there any non-zero function $f(n) $ such that $sum_{n=1}^infty frac{1} {f(n) +3^n}$ has an okay closed form?
    $endgroup$
    – Zacky
    Dec 2 '18 at 0:22








  • 1




    $begingroup$
    @Zacky There are some trivial choices like $f(n) = 2^n - 3^n$.
    $endgroup$
    – Daniel
    Dec 2 '18 at 0:33






  • 1




    $begingroup$
    There's something odd about this assignment. Part b) can be summed in closed form, but it takes techniques way beyond calc 2. Are you perhaps supposed to approximate the sums. I definitely second @TobyMak's advice: ask your teacher about it.
    $endgroup$
    – saulspatz
    Dec 2 '18 at 0:33






  • 3




    $begingroup$
    As noted by Carl, this question already exists ... math.stackexchange.com/q/1672121/442 but even that has no answer in nearly 3 years. No answer = cannot mark this as a duplicate.
    $endgroup$
    – GEdgar
    Dec 2 '18 at 1:14
















  • 2




    $begingroup$
    @FordDavis Ask your teacher about the question again. It's quite unlikely they expect for you to find the sum. Or maybe the question was $frac{1}{n3^n}$, which has a much nicer answer.
    $endgroup$
    – Toby Mak
    Dec 2 '18 at 0:21








  • 2




    $begingroup$
    What about this one: Is there any non-zero function $f(n) $ such that $sum_{n=1}^infty frac{1} {f(n) +3^n}$ has an okay closed form?
    $endgroup$
    – Zacky
    Dec 2 '18 at 0:22








  • 1




    $begingroup$
    @Zacky There are some trivial choices like $f(n) = 2^n - 3^n$.
    $endgroup$
    – Daniel
    Dec 2 '18 at 0:33






  • 1




    $begingroup$
    There's something odd about this assignment. Part b) can be summed in closed form, but it takes techniques way beyond calc 2. Are you perhaps supposed to approximate the sums. I definitely second @TobyMak's advice: ask your teacher about it.
    $endgroup$
    – saulspatz
    Dec 2 '18 at 0:33






  • 3




    $begingroup$
    As noted by Carl, this question already exists ... math.stackexchange.com/q/1672121/442 but even that has no answer in nearly 3 years. No answer = cannot mark this as a duplicate.
    $endgroup$
    – GEdgar
    Dec 2 '18 at 1:14










2




2




$begingroup$
@FordDavis Ask your teacher about the question again. It's quite unlikely they expect for you to find the sum. Or maybe the question was $frac{1}{n3^n}$, which has a much nicer answer.
$endgroup$
– Toby Mak
Dec 2 '18 at 0:21






$begingroup$
@FordDavis Ask your teacher about the question again. It's quite unlikely they expect for you to find the sum. Or maybe the question was $frac{1}{n3^n}$, which has a much nicer answer.
$endgroup$
– Toby Mak
Dec 2 '18 at 0:21






2




2




$begingroup$
What about this one: Is there any non-zero function $f(n) $ such that $sum_{n=1}^infty frac{1} {f(n) +3^n}$ has an okay closed form?
$endgroup$
– Zacky
Dec 2 '18 at 0:22






$begingroup$
What about this one: Is there any non-zero function $f(n) $ such that $sum_{n=1}^infty frac{1} {f(n) +3^n}$ has an okay closed form?
$endgroup$
– Zacky
Dec 2 '18 at 0:22






1




1




$begingroup$
@Zacky There are some trivial choices like $f(n) = 2^n - 3^n$.
$endgroup$
– Daniel
Dec 2 '18 at 0:33




$begingroup$
@Zacky There are some trivial choices like $f(n) = 2^n - 3^n$.
$endgroup$
– Daniel
Dec 2 '18 at 0:33




1




1




$begingroup$
There's something odd about this assignment. Part b) can be summed in closed form, but it takes techniques way beyond calc 2. Are you perhaps supposed to approximate the sums. I definitely second @TobyMak's advice: ask your teacher about it.
$endgroup$
– saulspatz
Dec 2 '18 at 0:33




$begingroup$
There's something odd about this assignment. Part b) can be summed in closed form, but it takes techniques way beyond calc 2. Are you perhaps supposed to approximate the sums. I definitely second @TobyMak's advice: ask your teacher about it.
$endgroup$
– saulspatz
Dec 2 '18 at 0:33




3




3




$begingroup$
As noted by Carl, this question already exists ... math.stackexchange.com/q/1672121/442 but even that has no answer in nearly 3 years. No answer = cannot mark this as a duplicate.
$endgroup$
– GEdgar
Dec 2 '18 at 1:14






$begingroup$
As noted by Carl, this question already exists ... math.stackexchange.com/q/1672121/442 but even that has no answer in nearly 3 years. No answer = cannot mark this as a duplicate.
$endgroup$
– GEdgar
Dec 2 '18 at 1:14












1 Answer
1






active

oldest

votes


















3












$begingroup$

What we can obtain, also by hand, is a reasonable estimation for example by



$$sum_{n=1}^inftyfrac{{1}}{n+3^n}approx 0.392<sum_{n=1}^3frac{{1}}{n+3^n}+sum_{n=4}^inftyfrac{{1}}{3^n}=$$$$=frac14+frac1{11}+frac1{30}+frac32-1-frac13-frac1{9}-frac1{27}approx 0.393$$






share|cite|improve this answer











$endgroup$













  • $begingroup$
    I dont remember us going over approximating the sum, but this might be what she wanted. Thanks for your help.
    $endgroup$
    – Ford Davis
    Dec 2 '18 at 2:40










  • $begingroup$
    @FordDavis Yes indeed in that case seems convenient use the fact that $3^n$ is larger than $n$ also for $n$ small to obtain a god estimation for the series.
    $endgroup$
    – gimusi
    Dec 2 '18 at 9:13











Your Answer





StackExchange.ifUsing("editor", function () {
return StackExchange.using("mathjaxEditing", function () {
StackExchange.MarkdownEditor.creationCallbacks.add(function (editor, postfix) {
StackExchange.mathjaxEditing.prepareWmdForMathJax(editor, postfix, [["$", "$"], ["\\(","\\)"]]);
});
});
}, "mathjax-editing");

StackExchange.ready(function() {
var channelOptions = {
tags: "".split(" "),
id: "69"
};
initTagRenderer("".split(" "), "".split(" "), channelOptions);

StackExchange.using("externalEditor", function() {
// Have to fire editor after snippets, if snippets enabled
if (StackExchange.settings.snippets.snippetsEnabled) {
StackExchange.using("snippets", function() {
createEditor();
});
}
else {
createEditor();
}
});

function createEditor() {
StackExchange.prepareEditor({
heartbeatType: 'answer',
autoActivateHeartbeat: false,
convertImagesToLinks: true,
noModals: true,
showLowRepImageUploadWarning: true,
reputationToPostImages: 10,
bindNavPrevention: true,
postfix: "",
imageUploader: {
brandingHtml: "Powered by u003ca class="icon-imgur-white" href="https://imgur.com/"u003eu003c/au003e",
contentPolicyHtml: "User contributions licensed under u003ca href="https://creativecommons.org/licenses/by-sa/3.0/"u003ecc by-sa 3.0 with attribution requiredu003c/au003e u003ca href="https://stackoverflow.com/legal/content-policy"u003e(content policy)u003c/au003e",
allowUrls: true
},
noCode: true, onDemand: true,
discardSelector: ".discard-answer"
,immediatelyShowMarkdownHelp:true
});


}
});














draft saved

draft discarded


















StackExchange.ready(
function () {
StackExchange.openid.initPostLogin('.new-post-login', 'https%3a%2f%2fmath.stackexchange.com%2fquestions%2f3022037%2fcalculating-sum-limits-n-1-infty-frac1n3n%23new-answer', 'question_page');
}
);

Post as a guest















Required, but never shown

























1 Answer
1






active

oldest

votes








1 Answer
1






active

oldest

votes









active

oldest

votes






active

oldest

votes









3












$begingroup$

What we can obtain, also by hand, is a reasonable estimation for example by



$$sum_{n=1}^inftyfrac{{1}}{n+3^n}approx 0.392<sum_{n=1}^3frac{{1}}{n+3^n}+sum_{n=4}^inftyfrac{{1}}{3^n}=$$$$=frac14+frac1{11}+frac1{30}+frac32-1-frac13-frac1{9}-frac1{27}approx 0.393$$






share|cite|improve this answer











$endgroup$













  • $begingroup$
    I dont remember us going over approximating the sum, but this might be what she wanted. Thanks for your help.
    $endgroup$
    – Ford Davis
    Dec 2 '18 at 2:40










  • $begingroup$
    @FordDavis Yes indeed in that case seems convenient use the fact that $3^n$ is larger than $n$ also for $n$ small to obtain a god estimation for the series.
    $endgroup$
    – gimusi
    Dec 2 '18 at 9:13
















3












$begingroup$

What we can obtain, also by hand, is a reasonable estimation for example by



$$sum_{n=1}^inftyfrac{{1}}{n+3^n}approx 0.392<sum_{n=1}^3frac{{1}}{n+3^n}+sum_{n=4}^inftyfrac{{1}}{3^n}=$$$$=frac14+frac1{11}+frac1{30}+frac32-1-frac13-frac1{9}-frac1{27}approx 0.393$$






share|cite|improve this answer











$endgroup$













  • $begingroup$
    I dont remember us going over approximating the sum, but this might be what she wanted. Thanks for your help.
    $endgroup$
    – Ford Davis
    Dec 2 '18 at 2:40










  • $begingroup$
    @FordDavis Yes indeed in that case seems convenient use the fact that $3^n$ is larger than $n$ also for $n$ small to obtain a god estimation for the series.
    $endgroup$
    – gimusi
    Dec 2 '18 at 9:13














3












3








3





$begingroup$

What we can obtain, also by hand, is a reasonable estimation for example by



$$sum_{n=1}^inftyfrac{{1}}{n+3^n}approx 0.392<sum_{n=1}^3frac{{1}}{n+3^n}+sum_{n=4}^inftyfrac{{1}}{3^n}=$$$$=frac14+frac1{11}+frac1{30}+frac32-1-frac13-frac1{9}-frac1{27}approx 0.393$$






share|cite|improve this answer











$endgroup$



What we can obtain, also by hand, is a reasonable estimation for example by



$$sum_{n=1}^inftyfrac{{1}}{n+3^n}approx 0.392<sum_{n=1}^3frac{{1}}{n+3^n}+sum_{n=4}^inftyfrac{{1}}{3^n}=$$$$=frac14+frac1{11}+frac1{30}+frac32-1-frac13-frac1{9}-frac1{27}approx 0.393$$







share|cite|improve this answer














share|cite|improve this answer



share|cite|improve this answer








edited Dec 2 '18 at 9:14

























answered Dec 2 '18 at 2:24









gimusigimusi

1




1












  • $begingroup$
    I dont remember us going over approximating the sum, but this might be what she wanted. Thanks for your help.
    $endgroup$
    – Ford Davis
    Dec 2 '18 at 2:40










  • $begingroup$
    @FordDavis Yes indeed in that case seems convenient use the fact that $3^n$ is larger than $n$ also for $n$ small to obtain a god estimation for the series.
    $endgroup$
    – gimusi
    Dec 2 '18 at 9:13


















  • $begingroup$
    I dont remember us going over approximating the sum, but this might be what she wanted. Thanks for your help.
    $endgroup$
    – Ford Davis
    Dec 2 '18 at 2:40










  • $begingroup$
    @FordDavis Yes indeed in that case seems convenient use the fact that $3^n$ is larger than $n$ also for $n$ small to obtain a god estimation for the series.
    $endgroup$
    – gimusi
    Dec 2 '18 at 9:13
















$begingroup$
I dont remember us going over approximating the sum, but this might be what she wanted. Thanks for your help.
$endgroup$
– Ford Davis
Dec 2 '18 at 2:40




$begingroup$
I dont remember us going over approximating the sum, but this might be what she wanted. Thanks for your help.
$endgroup$
– Ford Davis
Dec 2 '18 at 2:40












$begingroup$
@FordDavis Yes indeed in that case seems convenient use the fact that $3^n$ is larger than $n$ also for $n$ small to obtain a god estimation for the series.
$endgroup$
– gimusi
Dec 2 '18 at 9:13




$begingroup$
@FordDavis Yes indeed in that case seems convenient use the fact that $3^n$ is larger than $n$ also for $n$ small to obtain a god estimation for the series.
$endgroup$
– gimusi
Dec 2 '18 at 9:13


















draft saved

draft discarded




















































Thanks for contributing an answer to Mathematics Stack Exchange!


  • Please be sure to answer the question. Provide details and share your research!

But avoid



  • Asking for help, clarification, or responding to other answers.

  • Making statements based on opinion; back them up with references or personal experience.


Use MathJax to format equations. MathJax reference.


To learn more, see our tips on writing great answers.




draft saved


draft discarded














StackExchange.ready(
function () {
StackExchange.openid.initPostLogin('.new-post-login', 'https%3a%2f%2fmath.stackexchange.com%2fquestions%2f3022037%2fcalculating-sum-limits-n-1-infty-frac1n3n%23new-answer', 'question_page');
}
);

Post as a guest















Required, but never shown





















































Required, but never shown














Required, but never shown












Required, but never shown







Required, but never shown

































Required, but never shown














Required, but never shown












Required, but never shown







Required, but never shown







Popular posts from this blog

How do I know what Microsoft account the skydrive app is syncing to?

When does type information flow backwards in C++?

Grease: Live!